General solution of linear system

Click For Summary
The discussion revolves around finding the general solution of a linear system (M) with two equations and three unknowns, given a specific solution (2, -3, 1) and a corresponding homogeneous system (O) with solutions (-1, 1, 1) and (1, 0, 1). Participants note that the original poster's phrasing lacks clarity and completeness, particularly regarding the nature of the solutions. It is emphasized that the problem has infinitely many solutions, and a complete set of independent solutions is necessary to express any solution to the homogeneous system as a linear combination. The conversation highlights the importance of precise language in mathematical discussions, especially for those learning the concepts in a non-native language.
estro
Messages
239
Reaction score
0
I have this question, but don't know how to even start.

Suppose (M) is a linear system of 2 equations and 3 unknowns, where (2,-3,1) its solution.
Suppose (O) is a matching homogeneous linear system, where (-1,1,1) and (1,0,1) its solutions.

How can I find the general solution of (M)?
I'm totally lost with this one and appreciate any help.
 
Last edited:
Physics news on Phys.org
estro said:
I have this question, but don't know how to even start.

Suppose (M) is a linear system of 2 equations and 3 unknowns, where (2,-3,1).
Where (2, -3, 1) is what?
estro said:
Suppose (O) is a matching homogeneous system, where (-1,1,1) and (1,0,1) its solutions.
Is there a word missing here?
estro said:
How can I find the general solution of (M)?
I'm totally lost with this one and appreciate any help.
I understand what you're trying to say, but I would like you to rephrase things so that you have complete thoughts.
 
Edited the first post. Sorry for the missing words (I guess 24+ hours without sleep make me skip words).
 
Last edited:
estro said:
I have this question, but don't know how to even start.

Suppose (M) is a linear system of 2 equations and 3 unknowns, where (2,-3,1) is a [its] solution.
Suppose (O) is a matching homogeneous linear system, where (-1,1,1) and (1,0,1) are its solutions.
Still missing words. Also, you cannot say that these are the solutions to the homogeneous system nor that (2, -3, 1) is the solution to the original system. Such a problem has an infinite number of solutions. If you can find a complete set of "independent" solutions then you can write any solution to the homogenous system as a linear combination of them. What is given here is not sufficient to conclude that there isn't a third linearly independent but to get an answer to this, we must assume so. It would have been a lot better if you had simply copied the problem as it was given.

Anyway, assuming that any solution to the homogeneous system must be of the form A(-1, 1, 1)+ B(1, 0, 1) for some numbers A and B. Notice that L(A(-1, 1, 1)+ B(1, 0, 1))= AL(-1, 1, 1)+ BL((1, 0, 1))= A(0)+ B(0)= 0. Further, if x is a solution to the original system, if L(x)= y where y was the given "right side" of the system, then L(A(-1, 1, 1)+ B(1, 0, 1)+ x)= AL(-1, 1, 1)+ BL(1, 0, 1)+ L(x)= 0+ 0+ L(x)= y.

How can I find the general solution of (M)?
I'm totally lost with this one and appreciate any help.
 
HallsofIvy said:
Still missing words. Also, you cannot say that these are the solutions to the homogeneous system nor that (2, -3, 1) is the solution to the original system. Such a problem has an infinite number of solutions. If you can find a complete set of "independent" solutions then you can write any solution to the homogenous system as a linear combination of them. What is given here is not sufficient to conclude that there isn't a third linearly independent but to get an answer to this, we must assume so. It would have been a lot better if you had simply copied the problem as it was given.

Anyway, assuming that any solution to the homogeneous system must be of the form A(-1, 1, 1)+ B(1, 0, 1) for some numbers A and B. Notice that L(A(-1, 1, 1)+ B(1, 0, 1))= AL(-1, 1, 1)+ BL((1, 0, 1))= A(0)+ B(0)= 0. Further, if x is a solution to the original system, if L(x)= y where y was the given "right side" of the system, then L(A(-1, 1, 1)+ B(1, 0, 1)+ x)= AL(-1, 1, 1)+ BL(1, 0, 1)+ L(x)= 0+ 0+ L(x)= y.

Sorry for my grammar, English is not my native tongue.
Anyway thanks for the hint.
 
Question: A clock's minute hand has length 4 and its hour hand has length 3. What is the distance between the tips at the moment when it is increasing most rapidly?(Putnam Exam Question) Answer: Making assumption that both the hands moves at constant angular velocities, the answer is ## \sqrt{7} .## But don't you think this assumption is somewhat doubtful and wrong?

Similar threads

  • · Replies 2 ·
Replies
2
Views
2K
  • · Replies 6 ·
Replies
6
Views
2K
  • · Replies 2 ·
Replies
2
Views
990
  • · Replies 3 ·
Replies
3
Views
2K
  • · Replies 5 ·
Replies
5
Views
1K
Replies
1
Views
2K
Replies
12
Views
2K
  • · Replies 1 ·
Replies
1
Views
2K
  • · Replies 4 ·
Replies
4
Views
1K